LSAT and Law School Admissions Forum

Get expert LSAT preparation and law school admissions advice from PowerScore Test Preparation.

 Administrator
PowerScore Staff
  • PowerScore Staff
  • Posts: 8917
  • Joined: Feb 02, 2011
|
#84859
Complete Question Explanation

Flaw in the Reasoning - CE. The correct answer choice is (C).

Answer choice (A):

Answer choice (B):

Answer choice (C): This is the correct answer choice.

Answer choice (D):

Answer choice (E):

This explanation is still in progress. Please post any questions below!
User avatar
 teddykim100
  • Posts: 46
  • Joined: Jan 10, 2022
|
#98760
Hello,

since the author neglects the idea that say, people who love movies are more prone to own VCRs (aka neglecting an alternative explanation for the appearance of both events), wouldn't the answer choice be (D)? The author is switching a necessary and sufficient condition.
User avatar
 Paul Popa
PowerScore Staff
  • PowerScore Staff
  • Posts: 64
  • Joined: Sep 20, 2022
|
#98847
Hi Teddy,

While you are definitely correct that the author is neglecting an alternative explanation for both events, I respectfully disagree that the author is switching between a sufficient and a necessary condition. Rather, the author is making a correlation-causation error. At its core, the argument is saying that VCR owners go to the movies more than non-owners, therefore VCRs stimulates their owners to go the movies more. There is a clear correlation-causation error here. Just because VCR owners go the movies more often than non-owners doesn't mean the VCR itself is what stimulates them to go. Like you said, it could be that VCR owners are more likely to love movies than non-owners, and that love of movies is what led to the purchase of the VCR and the constant theatergoing.

(C) says this.

(D) This is describing a mistaken reversal, but the author doesn't do that here. The author doesn't have any set of if...then statements that they then mistakenly reverse in their conclusion. Remember that we need to see conditional language in an argument in order for an answer choice like this to be feasible. Hope this helps!

Get the most out of your LSAT Prep Plus subscription.

Analyze and track your performance with our Testing and Analytics Package.